Đến nội dung

Dung Du Duong nội dung

Có 285 mục bởi Dung Du Duong (Tìm giới hạn từ 06-06-2020)



Sắp theo                Sắp xếp  

#585426 Về dùng đường dẫn link

Đã gửi bởi Dung Du Duong on 27-08-2015 - 20:49 trong Hướng dẫn - Trợ giúp - Giải đáp thắc mắc khi sử dụng Diễn đàn

 Em cx thử luôn My profile




#558948 Tính xác suất tạo được 1 tam giác

Đã gửi bởi Dung Du Duong on 12-05-2015 - 18:42 trong Tổ hợp - Xác suất và thống kê - Số phức

sai chỗ nào ?

Trước hết không gian mẫu bằng $C_{100}^{3}$

Bạn còn chưa xét đến trường hợp: tổng 2 cạnh lớn hơn cạnh còn lại - chỗ mấu chốt :namtay 

Còn về phần "cân hay đều" chả quan trọng gì hết




#558950 Tính xác suất tạo được 1 tam giác

Đã gửi bởi Dung Du Duong on 12-05-2015 - 18:47 trong Tổ hợp - Xác suất và thống kê - Số phức

không cần phải thế, đề bài cho 100 đoạn phân biệt rồi

bạn chưa đọc kĩ đề bài




#561732 Tính xác suất tạo được 1 tam giác

Đã gửi bởi Dung Du Duong on 26-05-2015 - 19:40 trong Tổ hợp - Xác suất và thống kê - Số phức

Xét các bộ 3 số khác nhau trong tập $E=\{1,2,...,n\}$ lập thành các cạnh $a,b,c$ của một tam giác, trong đó $a<b<c$

Điều kiện sẽ là $c+1\le a+b\le 2c-3$

- Với $a+b=c+1$ ta lập được các bộ $(a,b)$ thỏa mãn là $(2,c-1),\;(3,c-2), ...$ tất cả $\left\lfloor\frac{c}{2}\right\rfloor-1$ bộ

- Với $a+b=c+2$ ta lập được các bộ $(a,b)$ thỏa mãn là $(3,c-1),\;(4,c-2), ...$ tất cả $\left\lfloor\frac{c+1}{2}\right\rfloor-2$ bộ

........

- Với $a+b=2c-3$ ta lập được bộ $(a,b)$ thỏa mãn là $(c-1,c-2)$ tất cả $\left\lfloor\frac{2c-4}{2}\right\rfloor-(c-3)$ bộ

Như vậy ứng với mỗi giá trị của $c$, ta lập được số bộ thỏa mãn là 

$\displaystyle\quad \sum_{k=c}^{2c-4}\left\lfloor\frac{k}{2}\right\rfloor-\frac{(c-2)(c-3)}{2}=\left\lfloor\frac{(2c-4)^2}{4}\right\rfloor-\left\lfloor\frac{(c-1)^2}{4}\right\rfloor-\frac{(c-2)(c-3)}{2}$

$=\frac{(c-1)(c-2)}{2}-\left\lfloor\frac{(c-1)^2}{4}\right\rfloor$

 

Cho $c$ chạy từ $1$ đến $n$, lấy tổng ta được kết quả

$\displaystyle S_n=\frac{n(n-1)(n-2)}{6}-\sum_{k=1}^{n-1}\left\lfloor\frac{k^2}{4}\right\rfloor$ 

$\displaystyle \quad=\frac{n(n-1)(n-2)}{6}-\frac{n(n-1)(2n-1)}{24}+\frac{1}{4}\left\lfloor\frac{n}{2}\right\rfloor$

$\displaystyle \quad=\frac{n(n-1)(2n-7)}{24}+\frac{1}{4}\left\lfloor\frac{n}{2}\right\rfloor$

 

Thay số, ta có $S_{100}=79625$

$C_{100}^3=161700$. Từ đó có được xác suất cần tính.

 

Nhận xét của (thầy?) chanhquocnghiem thật là hay!

1.

Lời giải cho câu hỏi 1 cũng rất chính xác! Ở đây tôi muốn nhấn mạnh và chú ý tới các bạn về sự nhầm lẫn khi lựa chọn không gian mẫu (nói riêng) và các bài toán tổ hợp nói chung đó là việc hiểu bản chất của việc lựa chọn.

Khi đề bài không yêu cầu về sự khác nhau giữa các phần tử thì ta phải hiểu là các phần tử có thể bằng nhau!

Vậy tại sao không gian mẫu không phải $n^3$ mà là $C_{n+2}^3$ ?

Đáp án $n^3$ xảy ra khi ta quan tâm đến vị trí (thứ tự) của các phần tử, trong khi đó bản chất của việc chọn ra 3 số xem có thỏa yêu cầu không thì đâu cần đến thứ tự?  $(a,a,b)$ với $(a,b,a)$ có gì khác nhau?

Như vậy KGM là số tổ hợp lặp chập $3$ của $n$ phần tử.

Có thể hiểu là số các bộ $(a,b,c)$ với $a,b,c\in\{1,2,...,n\}$ sao cho $a\le b\le c$. Bởi vì với $3$ số bất kỳ ta luôn sắp được trật tự trên

Như vậy thì $1\le a\le b\le c\le n\Leftrightarrow 1\le a<b+1<c+2\le n+2$

Và dễ thấy số bộ thỏa mãn như vậy là $C_{n+2}^3$

2.

Ta thấy rằng: vì $u_{n-2}+u_{n-1}=C_n^3$ và $u_n+u_{n+1}=C_{n+2}^3$ nên:

$u_{n-2}$ là số các tam giác khác nhau có 3 cạnh là các số nguyên dương đôi một khác nhau và không quá $n$

$u_{n-1}$ là số bộ 3 số nguyên dương đôi một khác nhau không quá $n$ đồng thời không lập được thành 3 cạnh của một tam giác

$u_n$ là số bộ 3 số nguyên dương (không nhất thiết khác nhau) không quá $n$ cũng không lập được thành 3 cạnh của một tam giác

$u_{n+1}$ là số các tam giác khác nhau có 3 cạnh là các số nguyên dương (không nhất thiết khác nhau) không quá $n$

 

1. Ba số tự nhiên được chọn có thể khác nhau từng đôi một hoặc có ít nhất 2 số giống nhau.

   + Nếu 3 số khác nhau từng đôi một thì có $C_{n}^{3}$ cách.

   + Nếu có ít nhất 2 số giống nhau thì có $n^2$ cách (chọn $2$ số giống nhau có $n$ cách, chọn số còn lại cũng có $n$ cách)

   $\Rightarrow$ số phần tử không gian mẫu là :

   $C_{n}^{3}+n^2=\frac{n(n-1)(n-2)}{6}+n^2=\frac{n(n+1)(n+2)}{6}=C_{n+2}^{3}$

 

2. Ta có :

   $u_{n-1}=u_{n-2}+\left \lfloor \frac{(n-1)^2}{4} \right \rfloor$

   $u_{n}=u_{n-2}+\left \lfloor \frac{(n-1)^2}{4} \right \rfloor+\left \lfloor \frac{n^2}{4} \right \rfloor$

   $u_{n+1}=u_{n-2}+\left \lfloor \frac{(n-1)^2}{4} \right \rfloor+\left \lfloor \frac{n^2}{4} \right \rfloor+\left \lfloor \frac{(n+1)^2}{4} \right \rfloor$

 

   $\Rightarrow \left ( u_{n-2}+u_{n+1} \right )-\left ( u_{n-1} +u_{n}\right )=\left \lfloor \frac{(n+1)^2}{4} \right \rfloor-\left \lfloor \frac{(n-1)^2}{4} \right \rfloor$

   Để ý rằng $\frac{(n+1)^2}{4}-\frac{(n-1)^2}{4}=n$ (là một số nguyên) nên ta có :

   $\left ( u_{n-2}+u_{n+1} \right )-\left ( u_{n-1}+u_{n} \right )=n$

Các thầy (?) tích cực quá, em cảm ơn vì những bài tập trên !!! ~O) 




#558946 Tính xác suất tạo được 1 tam giác

Đã gửi bởi Dung Du Duong on 12-05-2015 - 18:38 trong Tổ hợp - Xác suất và thống kê - Số phức

Một tam giác là có thể là tam giác cân,tam giác đều hoặc tam giác thường

Số cách chọn tam giác thường là $C_{100}^3$

Số cách chọn tam giác cân : có thể là tam giác cân nhọn hoặc tam giác cân tù nên số tam giác thỏa mãn là $2.C_{100}^2$

Số cách chọn tam giác đều là $C_{100}^1$

xác xuất để chọn ra 3 đoạn thẳng tạo được 1 tam giác là:

$\frac{C_{100}^1+2.C_{100}^2+C_{100}^3}{3^{100}}$ :))

Bạn làm sai rồi :namtay 




#558913 Tính xác suất tạo được 1 tam giác

Đã gửi bởi Dung Du Duong on 12-05-2015 - 15:59 trong Tổ hợp - Xác suất và thống kê - Số phức

Cho 100 đoạn thẳng phân biệt có độ dài là 1 số trong các số tự nhiên từ 1 đến 100. Tính xác suất để chọn ra 3 đoạn thẳng tạo được 1 tam giác




#574700 Chứng minh rằng : n chẵn

Đã gửi bởi Dung Du Duong on 22-07-2015 - 21:22 trong Số học

Cho : $\left\{\begin{matrix}n\in \mathbb{N} & & \\ & & \end{matrix}\right.7^{n}-3^{n}\vdots n$

 

a) Chứng minh rằng : n chẵn 

b)Tìm n 

 

 

 

:icon12:  :like  :dislike  :icon12:

 

Thế $n=1$ không thoả mãn à  :closedeyes:

Cho n > 1 bạn ơi :D  :D  :D




#574731 Chứng minh rằng : n chẵn

Đã gửi bởi Dung Du Duong on 22-07-2015 - 22:20 trong Số học

$7^n\equiv 7(modn);3^n\equiv 3(modn)\Rightarrow 7^n-3^n\equiv 2(modn)\Rightarrow n=2$

Vayaj $n=2$  là số chẵn :v

 

Mình thấy $n=4;n=8$ cũng thoả mãn mà nhỉ? :closedeyes:

Bạn Thao Huyen làm nhầm rồi (7,n) và (3,n) khác 1 nên ko áp dụng Phéc-ma được




#587866 CM vuông góc BM

Đã gửi bởi Dung Du Duong on 07-09-2015 - 20:52 trong Hình học

Cho tam giác ABC ngoại tiếp (O) với các tiếp điểm E,F,D lần lượt thuộc AB,AC,BC. Kẻ OC cắt EF tại M. CMR: CM vuông góc BM

Ngoài ra ta còn có 1 mở rộng nhỏ như sau:

Cho tam giác ABC ngoại tiếp (I) các tiếp điểm là D,E,F  thuộc AB,AC,BC . Gọi AO và CO lần lượt giao với EF và DE tại N,M (M,N thuộc đoạn DF và EF). CMR: tứ giác AMNC nội tiếp 




#587837 CM vuông góc BM

Đã gửi bởi Dung Du Duong on 07-09-2015 - 19:54 trong Hình học

Bạn nó như vậy là ko đúng, hình vẽ của mình là trong một trừờng hợp nhất định, nên nếu khi làm mình phải nói xét bài toán trong trường hợp hình vẽ đó, cá trường hợp còn lại tương tự, mình chưa nói nên sai, nhưng hình vẽ thì ko sai bạn nhé! Mình sẽ sửa lại bằng cách dùng góc định hướng, tư tưởng vẫn vậy, bạn xem lại thử nhé!

 

mình xin nói luôn: bạn sai ngay từ khâu vẽ hình nên sai hết rồi :(  :(  :(  :(  :(

Có bạn nào giúp mình với!!! :ukliam2:

 

Bài này là 1 chi tiết trong đề thi VN TST 2011 đó! Đọc lời giải mà mình ko hiểu tại sao tác giả lại nói rằng điều này hiển nhiên!

Mình cứ đợi mãi mà ........ Bài này thực ra hiển nhiên rất cơ bản

Bạn tự vẽ hình nhé: 

Có tam giác MFD cân tại M, tam giác BED cân tại B   (1)

Có góc MFD = góc BED                                            (2)

Từ (1) (2) ==> 2 tam giác trên đồng dạng ==> tứ giác BEMD nội tiếp ==> đpcm  :icon6:  :icon6:  :icon6:




#574713 Chứng minh rằng : n chẵn

Đã gửi bởi Dung Du Duong on 22-07-2015 - 21:39 trong Số học

Nếu thế thì chỉ có thể cm $n$ chẵn chứ không thể tìm cụ thể từng số $n$ vì $n=2k$ luôn thoả mãn giả thiết rồi  :icon6:

Bạn nhầm, với n=6 thì ko thỏa mãn đâu :closedeyes:




#588598 Newton và Einstein

Đã gửi bởi Dung Du Duong on 12-09-2015 - 21:11 trong Các nhà Toán học

Mình xin trình bày 1 trong nhiều sự khác nhau của Newton và Einstein: ~O) 

+Newton cho rằng (cx như những gì chúng ta đc học ở trường): Trái Đất quay quanh Mặt Trời vì lực hấp dẫn của Mặt Trời hút và khiến Trái Đất quay quanh nó. ~O) 

+Còn Einstein lại cho rằng: Do lực hấp dẫn của Mặt Trời quá lớn mà khiến cho không gian xung quanh nó bị bẻ cong, và trong khi Trái Đất quay quanh Mặt Trời thì thực chất nó đang đi thẳng trong 1 không gian bị uốn cong. Điều này giải thích cho việc Ánh sáng bị Lực hấp dẫn quá lớn của Hố đen bẻ cong đến mức bị hút vào trong và ko thể thoát ra khỏi nó (theo Thuyết tương đối) ~O) 

Hình gửi kèm

  • khoa-hoc-se-lam-sang-to-thuyet-tuong-doi-cua-albert-einstein.jpg
  • khoa-hoc-se-lam-sang-to-thuyet-tuong-doi-cua-albert-einstein (1).jpg



#576903 Tìm min A = $\frac{bc}{a(2b+c)}+\frac...

Đã gửi bởi Dung Du Duong on 30-07-2015 - 21:31 trong Bất đẳng thức và cực trị

Kí hiệu (0;2] nghĩa là j` ạ?

Nó có nghĩa là $0<x \leqslant2$ đó bạn




#560994 CMR: $\sum \frac{a^{3}}{2a^{2...

Đã gửi bởi Dung Du Duong on 22-05-2015 - 20:40 trong Bất đẳng thức - Cực trị

Em thấy đoạn cuối hoàn toán đúng, mà sao anh ra được cái này vậy.

Ban đầu mình cx nghĩ là đúng,cách rất là hay, nhưng mà khi tính lại vài lần thì phân tích nhân tử lại ra như thế, có thể là mình nhầm chăng!




#558658 CMR: $\sum \frac{a^{3}}{2a^{2...

Đã gửi bởi Dung Du Duong on 10-05-2015 - 20:25 trong Bất đẳng thức - Cực trị

BĐT như thế này thì tự hiểu nó là nguyên dương!

 

Đặt biểu thức vế trái là $A$ thì áp dụng Cauchy Schwarz ta có

 

$A=\sum \frac{a^4}{2a^3-a^2b+3ab^2}\geq \frac{(a^2+b^2+c^2)^2}{2(a^3+b^3+c^3)-\sum a^2b+3\sum ab^2}\geq \frac{a+b+c}{4}$

 

$\Leftrightarrow 4(a^2+b^2+c^2)^2\geq (a+b+c)\left [ 2(a^3+b^3+c^3)-\sum a^2b+3\sum ab^2 \right ]$

 

$\Leftrightarrow 2(a^2+b^2+c^2)^2\geq \sum a^3b+5\sum ab^3$ $(*)$

 

Có BĐT phụ quen thuộc sau: $(a^2+b^2+c^2)^2\geq 3(a^3b+b^3c+c^3a)$ hoặc $(a^2+b^2+c^2)^2\geq 3(ab^3+bc^3+ca^3)$

 

Sử dụng BĐT trên ta có ngay $(*)$ nên ta có đpcm.

 

----------------------------------------------------------------------------

Chứng minh BĐT phụ

 

Áp dụng bất đẳng thức $(x+y+z)^2\geq 3(xy+yz+xz)$ thì đặt $(x,y,z)=(a^2+bc-ab,b^2+ac-bc,c^2+ab-ac)$ ta có ngay đpcm

Mình nghĩ đoạn cuối cùng có vấn đề:

Liệu BĐT cuối cùng: (a-b)(b-c)(a-c)(a+b+c)$\geqslant 0$ với mọi a,b,c dương đc không!

Bạn thử tính kĩ lại xem




#558614 CMR: $\sum \frac{a^{3}}{2a^{2...

Đã gửi bởi Dung Du Duong on 10-05-2015 - 15:10 trong Bất đẳng thức - Cực trị

Bài toán thiếu điều kiện của $a,b,c$ thì phải. Cho $a=b=-9$ và $c=-5$ thì bất đẳng thức trên sai :D

Xin lỗi các bạn, mình viết sai đề bài




#558567 CMR: $\sum \frac{a^{3}}{2a^{2...

Đã gửi bởi Dung Du Duong on 10-05-2015 - 08:35 trong Bất đẳng thức - Cực trị

Cho a,b,c > 0. CMR: $\sum \frac{a^{3}}{2a^{2}-ab+3b^{2}} \geqslant \frac{a+b+c}{4}$




#560770 CMR: $\sum \frac{a^{3}}{2a^{2...

Đã gửi bởi Dung Du Duong on 21-05-2015 - 19:21 trong Bất đẳng thức - Cực trị

đúng đấy! Giả sử $b\geq c\geq a$ thì BĐT trên đúng

tớ cũng biết vậy, nhưng mà nếu như $b\geqslant a\geqslant c$ thì nó lại ko đc, nó ko có vai trò giống nhau




#560975 CMR: $\sum \frac{a^{3}}{2a^{2...

Đã gửi bởi Dung Du Duong on 22-05-2015 - 20:07 trong Bất đẳng thức - Cực trị

vai trò của các biến là bình đẳng như nhau

thế cậu giải thích sao về việc $b\geqslant a\geqslant c$ thì lại ko thỏa mãn!




#583335 x,y,z đồng quy tại 1 điểm nằm trên (O)

Đã gửi bởi Dung Du Duong on 20-08-2015 - 15:57 trong Hình học

Clipboardimage2015-08-20151413.png       

 Hạ $MI \perp AB$, $MJ \perp BC$, $MK \perp AC$. Giả sử $XY$ đối xứng với $A'B'$ qua $AB$.

Ta sẽ chứng minh $XY$ đi qua $M$

Ta có một số tính chất cơ bản sau:

- $A', B', C'$ thẳng hàng (đường thẳng $Steiner$)

- $I, J, K$ thẳng hàng (đường thẳng $Simson$)

Hai định lý trên có thể xem tại: https://julielltv.wo...-thang-steiner/

Vì $XY$ và $A'B'$ đối xứng nhau qua $AB$ nên $XY$, $A'B'$ và $AB$ đồng quy tại 1 điểm. Giả sử điểm đó là điểm $Z$.

Ta có: $\widehat{YZA'}= 2 \widehat{AZA'}= 2\widehat{C'ZI}$

Vì $C'$ đối xứng với $M$ qua $AB$ và $Z$ nằm trên $AB$ nên $ 2\widehat{C'ZI}= \widehat{C'ZM}$

Do đó $\widehat{YZA'}= \widehat{C'ZM}$

Mà $A',C',Z$ thẳng hàng nên $Y,Z,M$ thẳng hàng. Mà $X,Y,Z$ thẳng hàng nên $XY$ đi qua $M$ hay $x$ đi qua $M$.

Chứng minh tương tự $y,z$ cũng đi qua $M$

Vậy $x,y,z$ đồng quy tại $M$ nằm trên $(O)$. 

Thực chất dụng ý của mình cx giống như bạn 30 minutes đã nêu trên

Đường thẳng A',B',C' đi qua 1 điểm cố định là trực tâm tam giác ABC nên các đường thẳng đối xứng với nó mới đồng quy tại 1 điểm nằm trên (O)

Nhưng cách của bạn dễ hơn mình tưởng rất nhiều, thanks :lol:




#583647 x,y,z đồng quy tại 1 điểm nằm trên (O)

Đã gửi bởi Dung Du Duong on 21-08-2015 - 17:03 trong Hình học

Bạn có thể tham khảo ở tài liệu này

Bạn có biết cách CM điểm anti-Steiner khác ngoài cách dùng góc định hướng ko (nói thật là phần này mình chưa học)

Mình cx có tìm thấy 1 vài chuyên đề về nó nhưng mà toàn cách trên thôi




#583222 x,y,z đồng quy tại 1 điểm nằm trên (O)

Đã gửi bởi Dung Du Duong on 19-08-2015 - 22:35 trong Hình học

Lâu lâu mới ngó vô bài hình này, mình có mở rộng chút xíu: :like 

Cho tam giác ABC nội tiếp (O), lấy M thuộc cung nhỏ BC, lấy A', B', C' lần lượt đối xứng với M qua BC, AC, AB. Tiếp tục lấy các đường thẳng x,y,z lần lượt đối xứng với A'B', B'C', C'A' qua AB, AC, BC. CMR: x,y,z đồng quy tại 1 điểm nằm trên (O)  :like 




#583528 x,y,z đồng quy tại 1 điểm nằm trên (O)

Đã gửi bởi Dung Du Duong on 21-08-2015 - 09:34 trong Hình học

Bài này thì chỉ cần làm như vậy thôi còn với bài tổng quát khi đường thẳng bất kỳ đi qua trực tâm $H$ chính là định lý Collings

bạn cho mình cái links của định lí này đc ko, mình tìm mãi ko thấy




#593711 Đề thi HSG lớp 12 tỉnh Hải Dương 2015-2016 vòng 2

Đã gửi bởi Dung Du Duong on 14-10-2015 - 19:14 trong Thi HSG cấp Tỉnh, Thành phố. Olympic 30-4. Đề thi và kiểm tra đội tuyển các cấp.

Đề thi chọn đội tuyển dự thi HSG Quốc gia Tỉnh Hải Dương 2015-2016

 

(thời gian: 180 phút)

 

Câu I:(5 điểm)

   Cho dãy số {$y_n$} thỏa mãn: $y_{1}>0, y^2_{n+1}=y_{1}+y_{2}+...+y_{n}$ với $n \geq 1$. CMR: dãy số {$\frac{y_{n}}{n}$} có giới hạn và tìm giới hạn đó.

 

Câu II:(6 điểm)

   1, Cho tam giác ABC nội tiếp (O). Một đường tròn tiếp xúc với tia AB,AC lần lượt tại E và F, đồng thời tiếp xúc trong với đường tròn (O) tại T. Tiếp tuyến tại A và T với (O) cắt nhau tại K. Đường thẳng TE cắt (O) tại điểm M khác T. CMR: K,M,N thẳng hàng.

 

   2, Cho tứ giác ABCD nội tiếp (O) có đường kính AC vuông góc với BD tại điểm H. Gọi I,J,K,L lần lượt là hình chiếu vuông góc của H trên các đường thẳng AB,BC,CD,DA. Biết IK và JL đều không đi qua H. CMR: giao điểm của IK và JL nằm trên OH.

 

Câu III:(4 điểm)

   Cho số nguyên dương m và số nguyên tố p với p>m. CMR: số các số nguyên dương n sao cho đa thức $f(x)=mx^2 - (m + n - p)x + n$ có nghiệm hữu tỉ bằng số ước nguyên dương của m.

 

Câu IV:(5 điểm)

   Cho một dãy 2016 ô vuông kề nhau xếp thành một hàng dài. Có bao nhiêu cách điền các số 1,2,3,4,5 vào các ô vuông đó sao cho mỗi ô vuông chỉ điền một số và hiệu hai số trong hai ô kề nhau chỉ nhận giá trị là 1 hoặc -1.




#578866 Có bao nhiêu cách chia 100 cái kẹo giống nhau cho 30 người sao cho mỗi người...

Đã gửi bởi Dung Du Duong on 05-08-2015 - 20:26 trong Tổ hợp - Xác suất và thống kê - Số phức

Có bao nhiêu cách chia 100 cái kẹo giống nhau cho 30 người sao cho mỗi người có 1 cái kẹo

$C_{100}^{30}$ cách!